[Toán 11]chứng minh giùm mình cái định lí !

H

hcuitv

[TẶNG BẠN] TRỌN BỘ Bí kíp học tốt 08 môn
Chắc suất Đại học top - Giữ chỗ ngay!!

ĐĂNG BÀI NGAY để cùng trao đổi với các thành viên siêu nhiệt tình & dễ thương trên diễn đàn.

Chứng minh định lí: Nếu [TEX]\left|q \right|[/TEX]<1 thì lim [TEX]q^2[/TEX]=0.
Tiện thể giải hộ 2 bài cơ bản nha:
*chứng minh các dãy số với số hạng tổng quát sau đây có giới hạn 0:
a. [TEX](-1)^n[/TEX]/(n+5)
b.[TEX]\frac{1}{n(n+1)}[/TEX]
 
N

ngomaithuy93

Chứng minh định lí: Nếu [TEX]\left|q \right|[/TEX]<1 thì lim [TEX]q^2[/TEX]=0.
Theo tớ thì thế này: |q|<1 nên có thể biểu diễn q dưới dạng: [TEX]q=\frac{1}{n}[/TEX] với [TEX]n \in Z[/TEX]
Mà[TEX] lim \frac{1}{n}=0[/TEX] nên [TEX]limq=0[/TEX] \Rightarrow [TEX]\lim|q|=0[/TEX]
À cậu ơi, có phải c/m lim|q|= |lim q| ko cậu? :D
 
D

duonganh1012

Chứng minh định lí: Nếu [TEX]\left|q \right|[/TEX]<1 thì lim [TEX]q^2[/TEX]=0.
Tiện thể giải hộ 2 bài cơ bản nha:
*chứng minh các dãy số với số hạng tổng quát sau đây có giới hạn 0:
a. [TEX](-1)^n[/TEX]/(n+5)
b.[TEX]\frac{1}{n(n+1)}[/TEX]
phần a) dùng giới hạn kẹp
phần b) chia cả tử và mầu cho [TEX]n^2[/TEX]
:)>-
 
V

vungocthanhsp2

Theo tớ thì thế này: |q|<1 nên có thể biểu diễn q dưới dạng: [TEX]q=\frac{1}{n}[/TEX] với [TEX]n \in Z[/TEX]
Mà[TEX] lim \frac{1}{n}=0[/TEX] nên [TEX]limq=0[/TEX] \Rightarrow [TEX]\lim|q|=0[/TEX]
À cậu ơi, có phải c/m lim|q|= |lim q| ko cậu? :D

Trời ơi sai rồi em ơi@-)@-)@-)@-)@-)@-)@-)@-)@-)@-)@-)@-)@-)@-)@-)
Định lí có lẽ là như thế này:
Với [TEX]\left| q \right| < 1[/TEX]
Ta có kết quả sau: [TEX] {\lim }\limits_{n \to + \infty } {q^n} = 0[/TEX]
Cái này bám chặt vào định nghĩa giới hạn dãy số là chứng minh được
Nếu em muốn chứng minh cụ thể có thể gửi nick yahoo ta sẽ gửi cách chứng minh cho
Bởi vì tôi đánh công thức toán học trên diễn đàn không thạo lắm
 
Last edited by a moderator:
Top Bottom